- PowerScore Staff
- Posts: 5972
- Joined: Mar 25, 2011
- Tue Dec 20, 2011 12:00 am
#67515
Complete Question Explanation
Justify-PR. The correct answer choice is (B)
In this stimulus, Saunders discusses two proposed plans for dealing with a particular group of abandoned houses that everyone agrees was a safety threat. Some wanted demolition while others were in favor of rehabilitating the houses. Because the demolition strategy was successful, Saunders asserts, this proves that those in favor of rehabilitation were wrong. The problem with this logic is that the choice of one option (demolition) made the results of the other option impossible to assess (because once you tear down the houses, there's no way to test if rehabbing them would have worked).
The question stem asks for the principle that would decisively prove that one of the two options was superior.
Answer choice (A): The portion that eliminates this answer is at the end: "unless the building is believed to pose a threat to neighborhood safety,"and we know from the first sentence of the stimulus that everyone at last week's meeting agreed the houses posed a threat to the safety.
Answer choice (B): This is the correct answer choice. This answer proves definitively that rehabilitation should have been adopted.
Answer choice (C): This answer begins by stating, "If one of two proposals for renovating..." But, only one of the two proposals was for renovating. Further, we don't know the exact status of funding for both proposals.
Answer choice (D): First, there is no way to know if the houses were basically "sound," just that some people claimed they were sound. Second, an investigation might not change the original decision. Essentially, you are trying to prove that one decision was correct and the other was incorrect. Does this answer make that determination? No, even if you take demolition as problematic, this still does not prove that rehabilitation was the superior.
Answer choice (E): This answer only shows that maybe, but not definitely, that rehabilitation should have been chosen. Thus, it is not strong enough to meet the standard for a correct answer in this question.
Justify-PR. The correct answer choice is (B)
In this stimulus, Saunders discusses two proposed plans for dealing with a particular group of abandoned houses that everyone agrees was a safety threat. Some wanted demolition while others were in favor of rehabilitating the houses. Because the demolition strategy was successful, Saunders asserts, this proves that those in favor of rehabilitation were wrong. The problem with this logic is that the choice of one option (demolition) made the results of the other option impossible to assess (because once you tear down the houses, there's no way to test if rehabbing them would have worked).
The question stem asks for the principle that would decisively prove that one of the two options was superior.
Answer choice (A): The portion that eliminates this answer is at the end: "unless the building is believed to pose a threat to neighborhood safety,"and we know from the first sentence of the stimulus that everyone at last week's meeting agreed the houses posed a threat to the safety.
Answer choice (B): This is the correct answer choice. This answer proves definitively that rehabilitation should have been adopted.
Answer choice (C): This answer begins by stating, "If one of two proposals for renovating..." But, only one of the two proposals was for renovating. Further, we don't know the exact status of funding for both proposals.
Answer choice (D): First, there is no way to know if the houses were basically "sound," just that some people claimed they were sound. Second, an investigation might not change the original decision. Essentially, you are trying to prove that one decision was correct and the other was incorrect. Does this answer make that determination? No, even if you take demolition as problematic, this still does not prove that rehabilitation was the superior.
Answer choice (E): This answer only shows that maybe, but not definitely, that rehabilitation should have been chosen. Thus, it is not strong enough to meet the standard for a correct answer in this question.
Dave Killoran
PowerScore Test Preparation
Follow me on X/Twitter at http://twitter.com/DaveKilloran
My LSAT Articles: http://blog.powerscore.com/lsat/author/dave-killoran
PowerScore Podcast: http://www.powerscore.com/lsat/podcast/
PowerScore Test Preparation
Follow me on X/Twitter at http://twitter.com/DaveKilloran
My LSAT Articles: http://blog.powerscore.com/lsat/author/dave-killoran
PowerScore Podcast: http://www.powerscore.com/lsat/podcast/